Board review embriology questions

अब Quizwiz के साथ अपने होमवर्क और परीक्षाओं को एस करें!

Chaptwr 16 1. Melanocytes are found in which epidermal layer? (A) Stratum basale (B) Stratum corneum (C) Stratum granulosum (D) Stratum lucidum (E) Stratum spinosum

1. A. Melanocytes are found in the stratum basale, the deepest layer of the epidermis, at the dermoepidermal junction.

Chapter 15 1. One day a 9-year-old girl surprisingly announces to her mother, "Guess what, mommy, I'm not a girl; I'm a boy." The mother is shocked but does act on the comment. During the next few years, the mother notices some tomboyish behavior and difficulty in social adjustment at school. When the girl is 12 years old, puberty starts with a striking virilization of the external genitalia. The mother is extremely concerned and seeks medical attention. What is the most likely cause? (A) Male pseudo-intersexuality (B) Female pseudo-intersexuality (C) Congenital adrenal hyperplasia (D) Testicular feminization (E) Illegal use of anabolic steroids

1. A. Reduced levels of androgens during fetal development of an XY male fetus cause a feminization of the male external genitalia such that the baby can be phenotypically mistaken for female. Parents raise the XY male baby as a girl until puberty or other medical problems bring the child to medical attention.

Chapter 10 1. Pancreatic islets consist of alpha, beta, and delta cells, which secrete glucagon, insulin, and somatostatin, respectively. These cells are derived from (A) mesoderm (B) endoderm (C) ectoderm (D) neuroectoderm (E) neural crest cells

1. B. Pancreatic islets form as isolated clumps of cells that bud from endodermal tubules.

1. Which of the following time intervals best describes the maximum susceptibility period to teratogens? (A) Week 1 (B) Weeks 3-8 (C) Weeks 9-38 2. Which of the following time intervals best describes the resistant period to teratogens? (A) Week 1 (B) Weeks 3-8 (C) Weeks 9-38

1. B. The embryonic period (weeks 3-8) is the time when the embryo is most susceptible to teratogens because all organ morphogenesis occurs at this time. 2. A. Week 1 is the resistant period when the conceptus demonstrates the "all-or-none" phenomenon (i.e., the conceptus will either die as a result of the teratogen or survive unaffected).

1. A congenital diaphragmatic hernia may re- sult from failure of the (A) septum transversum to develop (B) pleuroperitoneal membranes to fuse in a normal fashion (C) pleuropericardial membrane to develop completely (D) dorsal mesentery of the esophagus to develop (E) body wall to form the peripheral part of the diaphragm

1. B. The formation of the diaphragm occurs through the fusion of tissue from four different sources. The pleuroperitoneal membranes normally fuse with the three other components during week 6 of development. Abnormal development or fusion of one or both of the pleuroperitoneal membranes causes a patent opening between the thorax and abdomen through which abdominal viscera may herniate.

Chapter 5 1. The most common interventricular septal defect (VSD) seen clinically is (A) persistent truncus arteriosus (B) membranous VSD (C) common ventricle (D) foramen secundum defect (E) premature closure of foramen ovale

1. B. The most common of all cardiac congenital malformations seen clinically are membranous VSDs. The membranous interventricular septum forms by the proliferation and fusion of tissue from three different sources: the right and left bulbar ridges and the atrioventricular (AV) cushions. Because of this complex formation, the probability of defects is very high.

Chapter 3 1. Which of the following components plays the most active role in invading the endometrium during blastocyst implantation? (A) Epiblast (B) Syncytiotrophoblast (C) Hypoblast (D) Extraembryonic somatic mesoderm (E) Extraembryonic visceral mesoderm

1. B. The syncytiotrophoblast plays the most active role in invading the endometrium of the mother's uterus. During the invasion, endometrial blood vessels and endometrial glands are eroded and a lacunar network is formed.

Chapter 11 1. A young mother brings her recently born infant into your office and complains that the infant gags and chokes after swallowing milk. A physical examination indicates excessive saliva and mucus around the mouth and nose, abdominal distention, pneumonitis, and radiographs indicate air in the infant's stomach. What is the most likely cause? (A) Hypertrophic pyloric stenosis (B) Tracheoesophageal fistula (C) Congenital lobar emphysema (D) Respiratory distress syndrome (E) Pulmonary hypoplasia

1. B. Tracheoesophageal fistula is an abnormal communication between the trachea and esophagus that results from an improper division of the foregut by the tracheoesophageal septum. It is generally associated with esophageal atresia and polyhydramnios.

1. Accessory ribs are most commonly found at- tached to the (A) cervical vertebrae (B) thoracic vertebrae (C) lumbar vertebrae (D) manubrium (E) sternebrae

1. C. Accessory ribs are most commonly attached to lumbar vertebrae. When present (incidence 0.5%-1%), a cervical rib is usually attached to the seventh cervical vertebra. Cervical ribs may compress the brachial plexus and subclavian vessels and cause superior thoracic outlet syndrome.

Chapter 14 1. The indifferent embryo begins phenotypic sexual differentiation during (A) week 3 of development (B) week 5 of development (C) week 7 of development (D) week 12 of development (E) week 20 of development

1. C. The embryo during weeks 1-6 remains in an indifferent or undifferentiated stage. The embryo begins phenotypic sexual differentiation during week 7.

Chapter 9 1. The surface ectoderm gives rise to which of the following structures? (A) Dilator pupillae muscle (B) Retina (C) Lens (D) Sclera (E) Choroid

1. C. The lens forms from the lens placode that is induced by the optic cup.

Chapter 13 1. When does the metanephros become functional? (A) At week 3 of development (B) At week 4 of development (C) At week 10 of development (D) Just before birth (E) Just after birth

1. C. The metanephros begins to form at week 5 and starts to function in the fetus at about week 10. The pronephros is not functional in humans. The mesonephros is the interim kidney, which functions until the metanephros is ready.

Chapter 6 1. During the later stages of pregnancy, mater- nal blood is separated from fetal blood by the (A) syncytiotrophoblast only (B) cytotrophoblast only (C) syncytiotrophoblast and cytotrophoblast (D) syncytiotrophoblast and fetal endothelium (E) cytotrophoblast and fetal endothelium

1. D. During the later stages of pregnancy, the placental membrane becomes very thin and con- sists of two layers—the syncytiotrophoblast and the fetal endothelium.

Chapter 4 1. Which germ layers are present at the end of week 3 of development (day 21)? (A) Epiblast only (B) Epiblast and hypoblast (C) Ectoderm and endoderm (D) Ectoderm, mesoderm, and endoderm (E) Epiblast, mesoderm, and hypoblast

1. D. During week 3 of development, the process of gastrulation, which establishes the three primary germ layers (ectoderm, intraembryonic mesoderm, and endoderm), occurs. The origin of all tissues and organs of the adult can be traced to one of these germ layers because these are whence they "germinate."

1. Which of the following arteries gives rise to the axis artery of the lower limb? (A) External iliac artery (B) Femoral artery (C) Profunda femoris artery (D) Umbilical artery (E) Inferior gluteal artery

1. D. Early in development, the umbilical artery gives rise to the axis artery.

1. A 20-year-old woman presents at the emer- gency department with severe abdominal pain on the right side with signs of internal bleeding. She indicated that she has been sexually active without contraception and missed her last menstrual period. Based on this information, which of the following dis- orders must be included as an option in the diagnosis? (A) Ovarian cancer (B) Appendicitis (C) Normal pregnancy (D) Ectopic tubal pregnancy (E) Toxemia of pregnancy

1. D. Ectopic tubal pregnancy must always be an option in the diagnosis when a woman in her reproductive years presents with such symptoms. Ninety percent of ectopic implantations occur in the uterine tube. Ectopic tubal pregnancies result in rupture of the uterine tube and internal hemorrhage, which presents a major threat to the woman's life. The uterine tube and embryo must be surgically removed. The symptoms may sometimes be confused with appendicitis.

Chapter 7 1. Which one of following basal ganglia is de- rived from the diencephalon? (A) Amygdaloid nucleus (B) Head of the caudate nucleus (C) Tail of the caudate nucleus (D) Globus pallidus (E) Putamen

1. D. The globus pallidus has its origin from the diencephalon. Neuroblasts from the subthalamus migrate into the telencephalic white matter to form the globus pallidus.

1. Which of the following arteries is one of the first branches to form from the axis artery? (A) Radial artery (B) Ulnar artery (C) Axillary artery (D) Median artery (E) Brachial artery

1. D. The median artery is one of the first branches to form from the axis artery. In the adult, the median artery does not persist and is probably reduced to a small unnamed vessel. This is why the median nerve does not have an accompanying artery in the adult like the ulnar nerve (ulnar artery) and radial nerve (radial artery).

1. Human chorionic gonadotropin (hCG) is produced by which of the following? (A) Ectoderm (B) Cytotrophoblast (C) Decidua basalis (D) Syncytiotrophoblast (E) Mesoderm

1. D. The syncytiotrophoblast produces hCG.

Chapter 12 1. The most common site of a thyroglossal cyst is (A) dorsal aspect of the neck (B) anterior border of the sternocleidomastoid muscle (C) superior mediastinum (D) midline close to the hyoid bone (E) base of the tongue

1. D. The thyroid gland forms from a diverticulum in the midline of the floor of the pharynx. The thyroid migrates caudally and passes ventral to the hyoid bone. During this migration, the thyroid remains connected to the tongue by the thyroglossal duct. If a part of the thyroglossal duct persists, a cyst will develop, usually near the hyoid bone.

Chapter 8 1. The cochlear duct contains the spiral organ of Corti and is derived from which of the following? (A) From both ectoderm and mesoderm (B) Neural crest (C) Endoderm (D) Mesoderm (E) Ectoderm

1. E. The cochlear duct is derived from a thickening of the surface ectoderm called the otic placode.

Study Questions for Chapter 18 1. The extrinsic eye muscles develop from which of the following? (A) Cervical somites (B) Epimere (C) Hypomere (D) Occipital somites (E) Preotic somites

1. E. The extrinsic eye muscles arise from the preotic somites (myotomes) found near the prochordal plate. Recent research indicates that the extrinsic eye muscles are derived from somitomeres 1-3, and 5.

10. The midgut loop normally herniates through the primitive umbilical ring into the extraembryonic coelom during week 6 of devel- opment. Failure of the intestinal loops to return to the abdominal cavity by week 11 results in the formation of (A) omphalocele (B) gastroschisis (C) anal agenesis (D) ileal diverticulum (E) intestinal stenosis

10. A. An omphalocele results when intestinal loops fail to return to the abdominal cavity. Instead, the intestinal loops remain in the umbilical cord covered by amnion.

10. Which of the following arterial malformations is very common in premature infants? (A) Patent ductus arteriosus (B) Coarctation of the aorta (C) Right aortic arch (D) Double aortic arch (E) Abnormal origin of the right subclavian artery

10. A. Patent ductus arteriosus (PDA) is very common in premature infants. Infants with birth weight less than 1750 grams typically have a PDA during the first 24 hours postnatally. PDA is more common in female infants than in male infants.

10. Which of the following conditions results from failure of the anterior neuropore to close? (A) Hydrocephalus (B) Anencephaly (C) Mongolism (D) Craniosynostosis (E) Meningoencephalocele

10. B. Failure of the anterior neuropore to close results in anencephaly. The brain fails to develop; no cranial vault is formed.

10. Pharyngeal groove 1 gives rise to the (A) internal auditory meatus (B) external auditory meatus (C) eustachian tube (D) cervical sinus (E) primary tympanic cavity

10. B. Pharyngeal groove 1 gives rise to the external auditory meatus.

10. In oogenesis, which of the following events occurs immediately following the completions of meiosis II? (A) Degeneration of the zona pellucida (B) Sperm penetration of the corona radiata (C) Formation of a female pronucleus (D) Appearance of the blastocyst (E) Completion of cleavage

10. C. The secondary oocyte is arrested in metaphase of meiosis II, and it will remain in this meiotic stage until fertilization occurs. Following fertilization, the secondary oocyte completes meio- sis II, forming a mature ovum and a polar body. The nucleus of the mature ovum is called the female pronucleus, which fuses with the male pronucleus to form a zygote.

10. A woman has her pregnancy suddenly terminated due to intrauterine fetal death. At autopsy, the fetus shows severe pallor, generalized edema, and hepatosplenomegaly. Which of the following would you suspect? (A) VATER (B) β-Thalassemia minor (C) β-Thalassemia major (D) Hydrops fetalis (E) VACTERL

10. D. Hydrops fetalis is the most severe form of α-thalassemia, and is a direct result of the lack or decreased synthesis of the α-globin chain of hemoglobin α2β2.

10. In which stage of lung maturation is the blood-air barrier established? (A) Embryonic period (B) Pseudoglandular period (C) Canalicular period (D) Terminal sac period (E) Alveolar period

10. D. The simple cuboidal epithelium within the terminal sacs differentiates into pneumocytes within the terminal sac period. The rapidly proliferating capillary network makes intimate contact with the terminal sacs, and the blood-air barrier is established with type I pneumo- cytes. These events take place in the terminal sac period, which runs from embryonic week 24 until birth.

10. A 32-year-old pregnant woman at 30 weeks of gestation comes to her physician because of excess weight gain in a 2-week period. Ultraso- nography reveals polyhydramnios. Which fetal abnormality is most likely responsible for the polyhydramnios? (A) Bilateral kidney agenesis (B) Umbilical cord knots (C) Velamentous placenta (D) Hypoplastic lungs (E) Esophageal atresia

10. E. Polyhydramnios is associated with the inability of the fetus to swallow because of esophageal atresia or anencephaly. Polyhydramnios can also result from absorption defects such as duode- nal atresia. The inability of the embryo to swallow the amniotic fluid means that the fluid can- not be absorbed into the fetal blood and removed by the placenta and passed into the maternal blood.

10. Immediately after birth of a boy, a moist, red protrusion of tissue is noted just superior to his pubic symphysis. After observation, urine drainage is noted from the upper lateral corners of this tissue mass. What is the diagnosis? (A) Pelvic kidney (B) Horseshoe kidney (C) Polycystic disease of the kidney (D) Urachal cyst (E) Exstrophy of the bladder

10. E. The moist, red tissue mass that is exposed to the exterior is actually the posterior wall of the urinary bladder. This is called exstrophy of the bladder and is caused when the anterior abdominal wall and anterior wall of the bladder fail to form. The ureters open onto the posterior wall; therefore, urine drainage is apparent.

10. In a detached retina, the site of detachment is found (A) within the outer plexiform layer (B) within the inner plexiform layer (C) between the inner nuclear layer and the outer nuclear layer (D) between the choriocapillaris and the pigment epithelial layer (E) between the pigment epithelial layer and the layer of outer segments of rods and cones

10. E. The site of retinal detachment is between the pigment epithelial layer and the layer of outer segments of rods and cones; this corresponds to the intraretinal space between the inner and outer layers of the optic cup. Retinal detachment occurs when fluid from the vitreous compart- ment passes through a retinal hole and separates the pigment epithelial layer from the layer of outer segments of rods and cones.

11. Kupffer cells present in the adult liver are derived from (A) mesoderm (B) endoderm (C) ectoderm (D) neuroectoderm (E) neural crest cells

11. A. Kupffer cells are actually macrophages and are derived from mesoderm. Hepatocytes and the epithelial lining of the intrahepatic biliary tree are derived from endoderm.

11. The specialized group of mesenchy- mal cells that aggregate to form blood is- lands centrally and primitive blood vessels peripherally are called (A) fibroblasts (B) cardiac progenitor cells (C) angioblasts (D) myoblasts (E) osteoblasts

11. B. The angioblasts are the mesenchymal cells that form blood vessels in embryonic development, as well as embryonic blood cells.

11. A 25-year-old pregnant woman at 17 weeks of gestation comes to her OB/GYN for a normal examination. During routine blood tests, her serum α-fetoprotein (AFP) concentration is found to be markedly decreased for her gesta- tional age. Which abnormality will the physi- cian need to rule out based on these low AFP levels? (A) Spina bifida (B) Anencephaly (C) Omphalocele (D) Down syndrome (E) Esophageal atresia

11. D. Reduced AFP levels are associated with Down syndrome. All of the other defects (neural tube defects such as spina bifida and anencephaly, omphalocele, and esophageal atresias) are associated with elevated AFP levels.

11. A physician monitoring a newborn infant's heart sounds using a stethoscope hears the characteristic murmur of a patent ductus arteriosus. How soon after birth should this murmur normally disappear? (A) 1-2 months (B) 1-2 weeks (C) 1-2 days (D) 1-2 hours (E) Immediately

11. D. The ductus arteriosus functionally closes within 1-2 hours after birth via smooth muscle contraction of the tunica media. Before birth, the patency of the ductus arteriosus is controlled by the low oxygen content of the blood flowing through it, which in turn stimulates production of prostaglandins, which cause smooth muscle to relax. After birth, the high oxygen content of the blood due to lung ventilation inhibits production of prostaglandins, causing smooth muscle contraction. Premature infants can be treated with prostaglandin synthesis inhibitors (such as indomethacin) to promote closure of the ductus arteriosus.

11. Which of the following structures is derived from the diencephalon? (A) Caudate nucleus (B) Cerebellum (C) Olfactory bulbs (D) Neurohypophysis (E) Adenohypophysis

11. D. The neurohypophysis develops from the diencephalon. The adenohypophysis (pars distalis, pars tuberalis, and pars intermedia) develops from Rathke pouch, an ectodermal diverticulum of the stomodeum. The caudate nucleus and olfactory bulbs develop from the telencephalon. The cerebellum develops from the metencephalon.

12. Caudal herniation of the cerebellar tonsils and medulla through the foramen magnum is called (A) Dandy-Walker syndrome (B) Down syndrome (C) Arnold-Chiari syndrome (D) cranium bifidum (E) myeloschisis

12. C. Arnold-Chiari syndrome is a cerebellomedullary malformation in which the caudal vermis and medulla herniate through the foramen magnum, resulting in communicating hydrocepha- lus. Arnold-Chiari syndrome is frequently associated with spina bifida.

12. The simple columnar and stratified colum- nar epithelia lining the lower part of the anal canal is derived from (A) mesoderm (B) endoderm (C) ectoderm (D) neuroectoderm (E) neural crest cells

12. C. The anal canal is formed from two components—the hindgut and proctodeum. The epithelium lining the lower anal canal is derived from ectoderm lining the proctodeum.

12. The epiblast is capable of forming which of the following germ layers? (A) Ectoderm only (B) Ectoderm and mesoderm only (C) Ectoderm and endoderm only (D) Ectoderm, mesoderm, endoderm (E) Mesoderm and endoderm only

12. D. The epiblast is capable of forming all three germ layers (ectoderm, mesoderm, and endo- derm) during gastrulation. Epiblast cells migrate to the primitive streak and invaginate into a space between the epiblast and hypoblast. Some of these epiblast cells displace the hypoblast to form the definitive endoderm. Migrating epiblast cells also form the intraembryonic mesoderm. The remaining epiblast cells form the ectoderm.

12. How soon after birth does the foramen ovale close? (A) 1-2 months (B) 1-2 weeks (C) 1-2 days (D) 1-2 hours (E) Immediately

12. E. The foramen ovale functionally closes almost immediately after birth as pressure in the right atrium decreases and pressure in the left atrium increases, thereby pushing the septum primum against the septum secundum. Anatomical fusion occurs much later in life; more than 25% of the population has probe patency of the foramen ovale, in which anatomical fusion does not occur.

13. A male newborn has a hemangioma on the left frontotemporal region of his face and scalp. The cells forming the hemangioma are derived from which of the following cell layers? (A) Ectoderm only (B) Mesoderm only (C) Endoderm only (D) Ectoderm and mesoderm (E) Endoderm and mesoderm

13. B. A hemangioma is a vascular tumor that can be present at birth in which the abnormal proliferation of blood vessels leads to a mass resembling a neoplasm. Hemangiomas are mesodermal in origin, in that they are formed by embryonic blood cells and the vascular endothelium formed by angioblasts.

13. A 9-year-old boy presents with complaints of numbness and tingling in both feet. Exami- nation reveals no pulse in the femoral artery, increased blood pressure in the arteries of the upper extremity, and enlarged intercostal veins. Which of the following abnormalities would be suspected? (A) Double aortic arch (B) Tetralogy of Fallot (C) Postductal coarctation of the aorta (D) Right aortic arch (E) Abnormal origin of the right subclavian artery

13. C. No pulse in the femoral artery, increased blood pressure in the arteries of the upper extrem- ity, enlarged intercostal veins, and numbness and tingling in both feet are clinical symptoms indicative of postductal coarctation of the aorta. Because of the constriction of the aorta, the blood supply to the lower extremity is compromised.

13. The flexure that develops between the met- encephalon and the myelencephalon is called the (A) cephalic flexure (B) mesencephalic flexure (C) pontine flexure (D) cerebellar flexure (E) cervical flexure

13. C. The pontine flexure develops between the metencephalon (pons) and the myelencephalon (medulla). The pontine flexure results in lateral expansion of the walls of the metencephalon and myelencephalon, stretching of the roof of the fourth ventricle, and widening of the floor of the fourth ventricle (rhomboid fossa).

13. A baby born to a young woman whose pregnancy was complicated by polyhydram- nios was placed in the intensive care unit because of repeated vomiting containing bile. The stomach was markedly distended, and only small amounts of meconium had passed through the anus. What is the most likely diagnosis? (A) Esophageal stenosis (B) Annular pancreas (C) Hypertrophic pyloric stenosis (D) Extrahepatic biliary atresia (E) Duodenal atresia

13. E. This baby is suffering from duodenal atresia at a level distal to the opening of the common bile duct. This causes a reflux of bile and its presence in the vomitus. The pregnancy was complicated by polyhydramnios because the duodenal atresia prevented passage of amniotic fluid into the intestines for absorption.

14. Which of the following statements best describes the sulcus limitans? (A) It is found in the interpeduncular fossa (B) It is located between the alar and basal plates (C) It separates the medulla from the pons (D) It separates the hypothalamus from the thalamus (E) It separates the neocortex from the allocortex

14. B. The sulcus limitans separates the sensory alar from the motor basal plates. It is found in the developing spinal cord and on the surface of the adult rhomboid fossa of the fourth ventricle. The bulbopontine sulcus (inferior pontine sulcus) separates the medulla from the pons. The hypothalamic sulcus separates the thalamus from the hypothalamus. The rhinal sulcus separates the neocortex from the allocortex.

14. A 42-year-old woman presents with complaints of severe headaches, blurred vision, slurred speech, and loss of muscle coordination. Her last pregnancy 5 years ago resulted in a hydatidiform mole. Laboratory results show a high hCG level. Which of the following conditions is a probable diagnosis? (A) Vasa previa (B) Placenta previa (C) Succenturiate placenta (D) Choriocarcinoma (E) Membranous placenta

14. D. After a hydatidiform mole, it is very important to assure that all the invasive trophoblastic tissue is removed. High levels of hCG are a good indicator of retained trophoblastic tissue because such tissue produces this hormone. In this case, the trophoblastic tissue has developed into a malignant choriocarcinoma and metastasized to the brain, causing her symptoms of headache, blurred vision, and so on.

14. Which structure is derived from the same embryonic primordium as the dorsal root ganglia? (A) Gonads (B) Kidney (C) Pineal gland (D) Liver (E) Adrenal medulla

14. E. Both the chromaffin cells of the adrenal medulla and the dorsal root ganglia are derived from neural crest cells.

14. The coronary sinus is derived from which of the following? (A) Truncus arteriosus (B) Bulbus cordis (C) Primitive ventricle (D) Primitive atrium (E) Sinus venosus

14. E. The coronary sinus is derived from the sinus venosus.

15. Which structure is derived from the same embryonic primordium as the kidney? (A) Gonads (B) Epidermis (C) Pineal gland (D) Liver (E) Adrenal medulla

15. A. Both the kidneys and the gonads are derived from intermediate mesoderm. This longitudi- nal dorsal ridge of mesoderm forms the gonadal ridge, which is involved with the formation of the future kidneys and gonads.

15. The conus arteriosus is derived from which of the following? (A) Truncus arteriosus (B) Bulbus cordis (C) Primitive ventricle (D) Primitive atrium (E) Sinus venosus

15. B. The smooth part of the right ventricle, known as the conus arteriosus, is derived from the bulbus cordis.

15. Myelinated preganglionic sympathetic neu- rons have their cell bodies in (A) Clarke column (B) substantia gelatinosa (C) intermediolateral cell column (D) intermediomedial cell column

15. C. Myelinated preganglionic sympathetic neurons have their cell bodies in the intermedio- lateral cell column of the lateral horn; this cell column extends from C8 to L1. Myelinated preganglionic parasympathetic neurons have their cell bodies in the sacral autonomic nucleus, from S2 to S4.

15. At what location does the amniotic cavity develop? (A) Between the cytotrophoblast and syncytiotrophoblast (B) Within the extraembryonic mesoderm (C) Between the endoderm and mesoderm (D) Within the hypoblast (E) Within the epiblast

15. E. The amniotic cavity develops within the epiblast, and it is a cavity that contains the embryo and amniotic fluid.

16. The proximal part of the aorta is derived from which of the following? (A) Truncus arteriosus (B) Bulbus cordis (C) Primitive ventricle (D) Primitive atrium (E) Sinus venosus

16. A. The proximal part of the aorta is derived from the truncus arteriosus.

16. At the end of week 2 of development (day 14), what is the composition of the embryonic disk? (A) Epiblast only (B) Epiblast and hypoblast (C) Ectoderm and endoderm (D) Ectoderm, mesoderm, and endoderm (E) Epiblast, mesoderm, and hypoblast

16. B. The embryoblast consists of the two distinct cell layers (epiblast and hypoblast) at the end of development week 2 (day 14) and forms the bilaminar embryonic disk.

16. The choroid plexus of the fourth ventricle is derived from the (A) alar plate (B) basal plate (C) floor plate (D) rhombic lip (E) roof plate

16. E. The roof plate and its pial covering give rise to the choroid plexus, which invaginates into the fourth ventricle. The alar plate gives rise to sensory neurons; the basal plate gives rise to motor neurons; the floor plate contains decussating fibers; the rhombic lips give rise to the cerebellum.

17. The trabeculated part of the right ventricle is derived from which of the following? (A) Truncus arteriosus (B) Bulbus cordis (C) Primitive ventricle (D) Primitive atrium (E) Sinus venosus

17. C. The trabeculated part of the right ventricle is derived from the primitive ventricle.

17. Tanycytes are found principally in the (A) area postrema (B) cerebral aqueduct (C) lateral ventricles (D) third ventricle (E) fourth ventricle

17. D. Tanycytes are modified ependymal cells, found principally in the third ventricle. Tanycytes transport substances from the CSF to the hypophyseal portal system.

18. Which of the following most accurately describes the herniation of meninges and brain tissue through a defect in occipital bone? (A) Cranium bifidum with meningoencephalocele (B) Cranium bifidum with meningohydroencephalocele (C) Cranium bifidum with meningocele (D) Arnold-Chiari syndrome (E) Dandy-Walker syndrome

18. A. Cranium bifidum with meningoencephalocele consists of the herniation of meninges and brain tissue through a defect in occipital bone.

18. Tricuspid atresia is a cardiac malformation that involves which of the following septa? (A) Aorticopulmonary septum (B) Atrial septum (C) Atrioventricular septum (D) Interventricular septum

18. C. Tricuspid atresia involves the atrioventricular septum.

19. Which of the following is the most common cause of congenital hydrocephalus? (A) Cranium bifidum with meningoencephalocele (B) Cranium bifidum with meningohydroencephalocele (C) Aqueductal stenosis (D) Arnold-Chiari syndrome (E) Dandy-Walker syndrome

19. C. The most common cause of congenital hydrocephalus is aqueductal stenosis.

19. A muscular VSD is a cardiac malformation that involves which of the following septa? (A) Aorticopulmonary septum (B) Atrial septum (C) Atrioventricular septum (D) Interventricular septum

19. D. Muscular VSD is caused by perforations in the muscular interventricular septum.

2. The femur develops from which of the following? (A) Somite mesoderm (B) Lateral plate mesoderm (C) Intermediate mesoderm (D) Extraembryonic mesoderm (E) Sclerotome mesoderm

2. B. All bones of the lower limb form from lateral plate mesoderm that condenses along the central axis of the lower limb bud.

2. The humerus develops from which of the following? (A) Somite mesoderm (B) Lateral plate mesoderm (C) Intermediate mesoderm (D) Extraembryonic mesoderm (E) Sclerotome mesoderm

2. B. All bones of the upper limb form from lateral plate mesoderm that condenses alone the central axis of the upper limb bud.

2. Which process establishes the three definitive germ layers? (A) Neurulation (B) Gastrulation (C) Craniocaudal folding (D) Lateral folding (E) Angiogenesis

2. B. Gastrulation establishes the three primary germ layers during week 3 of development. Neurulation is the process by which neuroectoderm forms the neural plate, which eventually folds to form the neural tube.

2. Within hours after birth, a baby whose mother is diabetic had a rising respiratory rate and labored breathing. The baby became cyanotic and died. Postmortem histological ex- amination revealed collapsed alveoli lined with eosinophilic material. What is the diagnosis? (A) Congenital emphysema (B) Respiratory distress syndrome (C) Cystic fibrosis (D) Tracheoesophageal fistula (E) Pulmonary carcinoma

2. B. Respiratory distress syndrome is common in premature infants and infants of diabetic mothers. It is caused by a deficiency or absence of surfactant. Collapsed alveoli and eosinophilic material consisting of fibrin (hyaline membrane) can be observed histologically, indicating associated hyaline membrane disease.

2. Taste sensation from the oral part (anterior two-thirds) of the tongue is predominantly car- ried by (A) trigeminal nerve (CN V) (B) chorda tympani branch of the facial nerve (CN VII) (C) glossopharyngeal nerve (CN IX) (D) superior laryngeal branch of the vagus nerve (CN X) (E) recurrent laryngeal branch of the vagus nerve (CN X)

2. B. Taste sensation from the mucosa for the oral part of the tongue is carried by the chorda tympani branch of the facial nerve (CN VII). This part of the tongue forms from pharyngeal arch 1, so the trigeminal nerve (CN V) will carry sensory innervation from the mucosa.

2. The middle ear cavity (A) is of mesodermal origin (B) develops from pharyngeal pouch 1 (C) develops from pharyngeal arch 1 (D) develops from pharyngeal arch 2 (E) develops from the otic vesicle

2. B. The middle ear cavity develops from pharyngeal pouch 1 as it evaginates to form the tubo- tympanic recess.

2. When does a secondary oocyte complete its second meiotic division to become a mature ovum? (A) At ovulation (B) Before ovulation (C) At fertilization (D) At puberty (E) Before birth

2. C. At ovulation, a secondary oocyte begins meiosis II, but this division is arrested at metaphase. The secondary oocyte will remain arrested in metaphase until a sperm penetrates it at fertiliza- tion. Therefore, the term "mature ovum'" is somewhat of a misnomer because it is a secondary oocyte that is fertilized, and, once fertilized, the new diploid cell is known as a zygote. If fertil- ization does not occur, the secondary oocyte degenerates.

2. A young black girl shows isolated patches of skin and hair that lack melanin pigment. In ad- dition, other skin lesions are observed that look suspiciously like a malignant melanoma. What is the most likely diagnosis? (A) Type I oculocutaneous albinism (B) Type II oculocutaneous albinism (C) Piebaldism (D) Ichthyosis (E) Psoriasis

2. C. Piebaldism is an autosomal dominant disorder and is basically a localized albinism.

2. A congenital diaphragmatic hernia most commonly occurs (A) on the right anteromedial side (B) on the right posterolateral side (C) on the left anteromedial side (D) on the left posterolateral side (E) anywhere on the left side

2. D. Congenital diaphragmatic hernias occur most commonly on the left posterolateral side. The pleuroperitoneal membrane on the right side closes before the left for reasons that are not clear. Consequently, the patency on the left side remains unclosed for a longer time. The portion of the diaphragm formed by the pleuroperitoneal membrane in the newborn is located posterolateral.

2. Failure of the choroid fissure to close results in (A) congenital detached retina (B) congenital aniridia (C) congenital aphakia (D) coloboma iridis (E) microphthalmos

2. D. Failure of the choroid (optic) fissure to close results in a cleft of the iris—a coloboma iridis. This defect may extend into the ciliary body, choroid, optic nerve, or retina. Congenital aphakia—absence of the lens—may result from defective development of the lens placode.

2. The most common cause of female pseudo- intersexuality is (A) a 46, XO genotype (B) a 47, XXY genotype (C) lack of androgen receptors (D) congenital adrenal hyperplasia (E) inadequate production of testosterone and Müllerian-inhibiting factor (MIF)

2. D. Female pseudo-intersex individuals have a 46, XX genotype. This condition is most commonly caused by congenital adrenal hyperplasia, in which the fetus produces excessive amounts of androgens. The high androgen level will masculinize the female genitalia.

2. Which of the following clinical signs would be most obvious on examination of a patient with either tetralogy of Fallot or transposition of the great vessels? (A) Sweaty palms (B) Lack of femoral artery pulse (C) Pulmonary hypertension (D) Cyanosis (E) Diffuse red rash

2. D. Marked cyanosis is a distinct clinical sign in both tetralogy of Fallot and transposition of the great vessels. Any congenital cardiac malformation that allows right-to-left shunting of blood is sometimes called cyanotic heart disease. Right-to-left shunting allows poorly oxygenated blood from the right side of the heart to mix with highly oxygenated blood on the left side of the heart. This causes decreased oxygen tension to peripheral tissues, leading to a characteristic blue tinge (cyanosis) and bulbous thickening of the fingers and toes (clubbing).

2. The anterior fontanelle is usually closed by (A) birth (B) age 6 months (C) age 18 months (D) age 2 years (E) age 5 years

2. D. The anterior fontanelle is usually closed by 2 years of age; the posterior and sphenoid fontanelles are usually closed by the end of 6 months of age.

2. A 2-month-old baby with severe jaundice also has dark-colored urine (deep yellow) and white clay-colored stool. Which of the following disorders might be suspected? (A) Esophageal stenosis (B) Annular pancreas (C) Hypertrophic pyloric stenosis (D) Extrahepatic biliary atresia (E) Duodenal atresia

2. D. The baby is suffering from extrahepatic biliary atresia, which results from failure of the bile ducts to recanalize during development. This prevents bile from entering the duodenum.

2. The tongue muscles develop from which of the following? (A) Cervical somites (B) Epimere (C) Hypomere (D) Occipital somites (E) Preotic somites

2. D. The tongue muscles (intrinsic and extrinsic) arise from the occipital somites (myotomes).

2. A urachal cyst is a remnant of the (A) urogenital sinus (B) urogenital ridge (C) cloaca (D) allantois (E) mesonephric duct

2. D. The upper end of the urogenital sinus is in patent communication with the allan- tois, which lies in the umbilical cord. The allantois normally regresses and forms a fibrous cord. If a remnant persists, it forms a urachal cyst or sinus.

2. When are the axons of the corticospinal tracts fully myelinated? (A) In the late embryonic period (B) In the midfetal period (C) At birth (D) By the end of the first postnatal year (E) By the end of the second postnatal year

2. E. Axons of the corticospinal tracts are fully myelinated by the end of the second postnatal year; Babinski sign (extensor plantar reflex) is usually not elicitable before myelination of the cortico- spinal tracts.

2. The indifferent embryo completes phenotypic sexual differentiation during (A) week 3 of development (B) week 5 of development (C) week 7 of development (D) week 12 of development (E) week 20 of development

2. E. By week 12, female and male characteristics can be recognized. By week 20, phenotypic sexual differentiation is complete.

2. A reliable index of fetal-placenta function is maternal urinary level of (A) estrone (B) human placental lactogen (hPL) (C) prolactin (PRL) (D) progesterone (E) estriol

2. E. Maternal urinary levels of estriol have long been recognized as a reliable index of fetal-placental function because estriol production is dependent on a normal functioning fetal adrenal cortex, fetal liver, and placenta.

2. Between which two layers is the extraembry- onic mesoderm located? (A) Epiblast and hypoblast (B) Syncytiotrophoblast and cytotrophoblast (C) Syncytiotrophoblast and endometrium (D) Exocoelomic membrane and syncytiotrophoblast (E) Exocoelomic membrane and cytotrophoblast

2. E. The extraembryonic mesoderm is derived from the epiblast and is located between the exocoelomic membrane and the cytotrophoblast. The overall effect is to completely separate the embryoblast from the trophoblast, with the extraembryonic mesoderm serving as a conduit (connection) between them.

2. The maternal and fetal components of the placenta are (A) decidua basalis and secondary chorionic villi (B) decidua capsularis and secondary chori- onic villi (C) decidua parietalis and tertiary chorionic villi (D) decidua capsularis and villous chorion (E) decidua basalis and villous chorion

2. E. The placenta is a unique organ, in that it is a composite of tissue from two different sources— the mother and fetus. The maternal component is the decidua basalis, and the fetal component is the villous chorion.

20. Tetralogy of Fallot is a cardiac malformation that involves which of the following septa? (A) Aorticopulmonary septum (B) Atrial septum (C) Atrioventricular septum (D) Interventricular septum

20. A. Tetralogy of Fallot involves the aorticopulmonary septum.

20. Which of the following is associated with atresia of the foramen of Magendie and foram- ina of Luschka? (A) Cranium bifidum with meningoencephalocele (B) Cranium bifidum with meningohydroencephalocele (C) Aqueductal stenosis (D) Arnold-Chiari syndrome (E) Dandy-Walker syndrome

20. E. Dandy-Walker syndrome is congenital hydrocephalus associated with atresia of the outlet foramina of Magendie and Luschka. It is associated with agenesis of the cerebellar vermis and agenesis of the splenium of the corpus callosum.

21. d-Transposition of the great arteries is a cardiac malformation that involves which of the following septa? (A) Aorticopulmonary septum (B) Atrial septum (C) Atrioventricular septum (D) Interventricular septum

21. A. d-Transposition involves the aorticopulmonary septum.

21. Which of the following is associated with platybasia and malformation of the occipitovertebral joint? (A) Cranium bifidum with meningoencephalocele (B) Cranium bifidum with meningohydroencephalocele (C) Aqueductal stenosis (D) Arnold-Chiari syndrome (E) Dandy-Walker syndrome

21. D. Arnold-Chiari syndrome, a common congenital malformation, is frequently associated with platybasia and malformation of the occipitovertebral joint; other anomalies frequently seen are beaking of the tectum, aqueductal stenosis, kinking and herniation of the medulla, and herniation of the cerebellar vermis through the foramen magnum. Meningomyelocele (spina bifida) is a common component of the syndrome.

22. A 22-year-old pregnant woman at 20 weeks of gestation comes to her ON/GYN for a normal examination. During routine blood tests, her serum α-fetoprotein (AFP) concentration is markedly increased for her gestational age. Ul- trasonography reveals spina bifida in the fetus. At what week of gestation did this defect most likely occur? (A) 1-2 (B) 4-6 (C) 9-11 (D) 12-15 (E) 16-19

22. B. The posterior neuropore closes during week 4 (day 27). Failure of the posterior neuropore to close results in lower neural tube defects, such as spina bifida.

22. An insufficient amount of AV cushion mate- rial will result in which of the following? (A) Persistent truncus arteriosus (PTA) (B) Ebstein anomaly (C) Transposition of the great arteries (D) Common ventricle (E) Tricuspid atresia

22. E. Insufficient amount of AV cushion material will cause tricuspid atresia.

23. A partial development of the aorticopulmonary septum will result in which of the following? (A) Persistent truncus arteriosus (PTA) (B) Ebstein anomaly (C) Transposition of the great arteries (D) Common ventricle (E) Tricuspid atresia

23. A. Partial development of the aorticopulmonary septum will cause persistent truncus arteriosus.

23. Which structure is derived from the cranial neural crest cells? (A) Lens of the eye (B) Pia mater (C) Dura mater (D) Pineal gland (E) Olfactory placode, CN 1

23. B. The pia mater is the only listed structure that is derived from cranial neural crest cells

24. A failure of the tricuspid leaflets to attach to the annulus fibrosus will result in which of the following? (A) Persistent truncus arteriosus (PTA) (B) Ebstein anomaly (C) Transposition of the great arteries (D) Common ventricle (E) Tricuspid atresia

24. B. Failure of fusion of the tricuspid leaflets with the annulus fibrosus results in Ebstein anomaly.

25. A faulty fusion of the right and left bulbar ridges and AV cushion will result in which of the following? (A) Persistent truncus arteriosus (PTA) (B) Ebstein anomaly (C) Transposition of the great arteries (D) Common ventricle (E) Membranous VSD

25. E. Faulty fusion of the right and left bulbar ridges and AV cushions will cause membranous VSD.

26. The superior mesenteric artery is derived from which of the following? (A) Posterolateral arteries (B) Lateral arteries (C) Ventral arteries

26. C. The superior mesenteric artery is derived from ventral branches of the dorsal aorta, specifi- cally the vitelline arteries.

27. The arteries to the upper extremity are de- rived from which of the following? (A) Posterolateral arteries (B) Lateral arteries (C) Ventral arteries

27. A. Arteries to the upper extremity are derived from posterolateral branches of the dorsal aorta.

28. The gonadal arteries are derived from which of the following? (A) Posterolateral arteries (B) Lateral arteries (C) Ventral arteries

28. B. The gonadal arteries are derived from lateral branches of the dorsal aorta.

29. The proximal part of the internal carotid ar- tery is derived from which of the following? (A) Aortic arch 1 (B) Aortic arch 2 (C) Aortic arch 3 (D) Aortic arch 4 (E) Aortic arch 6

29. C. The proximal part of the internal carotid artery is derived from aortic arch 3.

3. The most common viral infection of fetus is (A) cytomegalovirus (B) rubella virus (C) herpes virus type 2 (D) varicella zoster virus (E) HIV

3. A. Cytomegalovirus (CMV) is the most common fetal infection and is the cause of cytomegalic inclusion disease.

3. During week 2 of development, the embryo- blast receives its nutrients via (A) diffusion (B) osmosis (C) reverse osmosis (D) fetal capillaries (E) yolk sac nourishment

3. A. During week 2 of development, the embryoblast receives its nutrients from endometrial blood vessels, endometrial glands, and decidual cells via diffusion. Diffusion of nutrients does not pose a problem, given the small size of the blastocyst during week 2. Although the begin- nings of a uteroplacental circulation are established during week 2, no blood vessels have yet formed in the extraembryonic mesoderm to carry nutrients directly to the embryoblast (this occurs in week 3).

3. A young infant shows extremely stretchable and fragile skin, hypermobile joints, and cigarette-paper scars over the knees. What is the most likely diagnosis? (A) Ehlers-Danlos syndrome (B) Junctional epidermolysis bullosa (C) Psoriasis (D) Ichthyosis (E) Piebaldism

3. A. Ehlers-Danlos syndrome is an autosomal dominant disorder involving the gene for peptidyl lysine hydroxylase.

3. The rectus femoris muscle develops from which of the following? (A) Posterior condensation (B) Anterior condensation (C) Lateral plate mesoderm (D) Extraembryonic mesoderm (E) Sclerotome mesoderm

3. A. Somite mesoderm migrates into the limb bud and forms two condensations. The posterior condensation of the lower limb gives rise to the extensors of the lower limb, which attain an anterior location in the adult because of the medial rotation of 90°.

3. The long head of the triceps muscle develops from which of the following? (A) Posterior condensation (B) Anterior condensation (C) Lateral plate mesoderm (D) Extraembryonic mesoderm (E) Sclerotome mesoderm

3. A. Somite mesoderm migrates into the limb bud and forms two condensations. The posterior condensation of the upper limb gives rise to the extensors of the upper limb, which attain a posterior location in the adult because of the lateral rotation of 90°.

3. Which of the following represents the general somatic efferent (GSE) column of the pons? (A) Abducent nucleus (B) Nucleus ambiguus (C) Hypoglossal nucleus (D) Inferior olivary nucleus (E) Inferior salivatory nucleus

3. A. The abducent nucleus represents the general somatic efferent (GSE) column of the pons.

3. The first indication of gastrulation in the embryo is (A) formation of the primitive streak (B) formation of the notochord (C) formation of the neural tube (D) formation of extraembryonic mesoderm (E) formation of tertiary chorionic villi

3. A. The formation of the primitive streak on the dorsal surface of the bilaminar embryonic disk is the first indication of gastrulation.

3. A congenital diaphragmatic hernia is usually life threatening because it is associated with (A) pulmonary hypoplasia (B) pulmonary hyperplasia (C) physiological umbilical hernia (D) liver hypoplasia (E) liver agenesis

3. A. The herniation of abdominal contents into the pleural cavity compresses the developing lung bud, resulting in pulmonary hypoplasia. Lung development on the ipsilateral (left) side of the herniation is most commonly affected, but lung development on the contralateral (right) side can also be compromised. The lungs may achieve normal size and function after surgical reduction of the hernia and repair of the diaphragm. However, mortality is high due to pulmonary hypoplasia.

3. The intervillous space of the placenta contains (A) maternal blood (B) fetal blood (C) maternal and fetal blood (D) amniotic fluid (E) maternal blood and amniotic fluid

3. A. The intervillous space contains only maternal blood as the spiral arteries of the endome- trium penetrate the outer cytotrophoblast shell.

3. Which of the following congenital cardiovas- cular malformations is most commonly associ- ated with maternal rubella infection? (A) Isolated dextrocardia (B) Patent ductus arteriosus (C) Persistent truncus arteriosus (D) Coarctation of the aorta (E) Double aortic arch

3. B. Patent ductus arteriosus (PDA) is the most common congenital cardiac malformation asso- ciated with rubella infection of the mother. It is unclear how the rubella virus acts to cause PDA.

3. The trachea is lined with pseudostratified ciliated columnar epithelium with goblet cells. This epithelium is derived from (A) neuroectoderm (B) endoderm (C) ectoderm (D) visceral mesoderm (E) mesoderm of fourth and sixth pharyngeal arches

3. B. The epithelial lining of the entire respiratory system (from tracheal epithelium to type I pneumocytes lining alveoli) is derived from endoderm.

3. The first fetal movements occur in which of the following trimesters? (A) First trimester (B) Second trimester (C) Third trimester

3. B. The first fetal movements occur in the second trimester.

3. The optic cup is an evagination of which of the following? (A) Telencephalon (B) Diencephalon (C) Mesencephalon (D) Metencephalon (E) Myelencephalon

3. B. The optic cup and its derivatives—the retina and optic nerve—develop from the diencephalon.

3. The otic vesicle (A) gives rise to the bony labyrinth (B) is found adjacent to the rhombencephalon (C) is derived from neuroectoderm (D) gives rise to the auricle (pinna) (E) gives rise to the tympanic membrane

3. B. The otic vesicle arises from an invagination of the surface ectoderm called the otic placode. The otic vesicle is found adjacent to the rhombencephalon.

3. The biceps brachii muscle develops from which of the following? (A) Hypomere (B) Epimere (C) Anterior condensation (D) Posterior condensation (E) Preotic somites

3. C. Because the biceps brachii muscle is a flexor of the antebrachium (forearm), it develops from the anterior condensation of myotomic mesoderm.

3. The most common cause of male pseudo- intersexuality is (A) a 45, XO genotype (B) a 47, XXY genotype (C) inadequate production of testosterone and MIF (D) congenital adrenal hyperplasia (E) lack of androgen receptors

3. C. Male pseudo-intersex individuals have a 46, XY genotype. This condition is most commonly caused by inadequate production of testosterone and MIF by the fetal testes. The low testosterone and MIF levels will stunt the development of the male genitalia.

3. A 28-day-old baby is brought to the physician because of projectile vomiting after feeding. Until this time, the baby has had no problems in feeding. On examination, a small knot is palpated at the right costal margin. Which of the following disorders might be suspected? (A) Esophageal stenosis (B) Annular pancreas (C) Hypertrophic pyloric stenosis (D) Extrahepatic biliary atresia (E) Duodenal atresia

3. C. The baby is suffering from hypertrophic pyloric stenosis. This occurs when the smooth muscle in the pyloric region of the stomach hypertrophies and obstructs passage of food. The hypertrophied muscle can be palpated at the right costal margin. The exact cause of this condition is not known.

3. After the sinovaginal bulbs have proliferated and fused, they form a solid core of endodermal cells called the (A) vestibule of the vagina (B) uterovaginal primordium (C) urogenital sinus (D) vaginal plate (E) clitoris

3. D. The sinovaginal bulbs proliferate, fuse, and form the vaginal plate under the inductive influence of the paramesonephric ducts. The vaginal plate then canalizes to form the inferior two thirds of the vagina.

3. The condition where the pedicles of the vertebral arches fail to fuse with the vertebral body is called (A) block vertebrae (B) cleft vertebrae (C) hemivertebrae (D) spondylolisthesis (E) spina bifida occulta

3. D. When the pedicles fail to fuse with the vertebral body, a condition called spondylolisthesis results. This allows the vertebral body to move anterior with respect to the vertebrae below it, causing lordosis.

3. How soon after fertilization occurs within the uterine tube does the blastocyst begin implantation? (A) Within minutes (B) By 12 hours (C) By day 1 (D) By day 2 (E) By day 7

3. E. The blastocyst begins implantation by day 7 after fertilization.

3. The intermaxillary segment forms via the fusion of the (A) maxillary prominences (B) mandibular prominences (C) palatine shelves (D) lateral nasal prominences (E) medial nasal prominences

3. E. The intermaxillary segment, which plays a critical role in the formation of the definitive adult palate, forms when the two medial nasal prominences fuse in the midline.

3. During surgery for a benign cyst on the kidney, the surgeon notes that the patient's right kidney has two ureters and two renal pelves. This malformation is (A) an abnormal division of the pronephros (B) an abnormal division of the mesonephros (C) formation of an extra mass of intermediate mesoderm (D) a premature division of the metanephric blastema (E) a premature division of the ureteric bud

3. E. The ureteric bud seems to be preprogrammed to undergo repeated divisions. These divisions normally begin on contact with the metanephric blastema. If the ureteric bud undergoes division prematurely, duplication of the ureter and renal pelvis occurs. In some circumstances, two separate kidneys may form.

30. A portion of the arch of the aorta is derived from which of the following? (A) Aortic arch 1 (B) Aortic arch 2 (C) Aortic arch 3 (D) Aortic arch 4 (E) Aortic arch 6

30. D. Part of the arch of the aorta is derived from aortic arch 4.

31. The proximal part of the right subclavian artery is derived from which of the following? (A) Aortic arch 1 (B) Aortic arch 2 (C) Aortic arch 3 (D) Aortic arch 4 (E) Aortic arch 6

31. D. The proximal part of the right subclavian artery is derived from aortic arch 4.

32. The portal vein is derived from which of the following? (A) Vitelline veins (B) Umbilical veins (C) Anterior cardinal veins (D) Posterior cardinal veins (E) Subcardinal veins

32. A. The portal vein is derived from the right vitelline vein.

33. The renal veins are derived from which of the following? (A) Vitelline veins (B) Umbilical veins (C) Anterior cardinal veins (D) Posterior cardinal veins (E) Subcardinal veins

33. E. The renal veins are derived from the subcardinal veins.

34. The superior mesenteric vein is derived from which of the following? (A) Vitelline veins (B) Umbilical veins (C) Anterior cardinal veins (D) Posterior cardinal veins (E) Subcardinal veins

34. A. The superior mesenteric vein is derived from the vitelline veins.

35. Closure of the foramen primum re- sults from fusion of which of the following structures? (A) Septum secundum and the fused atrioven- tricular cushions (B) Septum secundum and the septum primum (C) Septum primum and the fused atrioven- tricular cushions (D) Septum primum and the septum spurium (E) Septum primum and the sinoatrial valves

35. C. The foramen primum forms between the free edge of the septum primum and the atrioven- tricular (AV) cushions. It is closed when the septum primum fuses with the AV cushions.

36. A 3-day-old boy delivered at 32 weeks of gestation is experiencing respiratory distress syndrome. The physician detects a heart murmur characteristic of a patent ductus ar- teriosus, a diagnosis that is confirmed with an echocardiogram. Which embryonic structure is involved in this diagnosis? (A) Left third aortic arch (B) Right third aortic arch (C) Left sixth aortic arch (D) Umbilical arteries (E) Vitelline arteries

36. C. Patent ductus arteriosus (PDA) is a condition in which the ductus arteriosus, a blood ves- sel that allows blood to bypass the baby's lungs before birth, fails to normally close after birth. The ductus arteriosus is derived from the distal portion of the left sixth aortic arch

4. Which of the following arteries supplies foregut derivatives of the digestive system? (A) Celiac trunk (B) Superior mesenteric artery (C) Inferior mesenteric artery (D) Right umbilical artery (E) Intercostal artery

4. A. The artery that supplies foregut derivatives of the digestive system is the celiac trunk. The celiac trunk consists of the left gastric artery, splenic artery, and common hepatic artery. The superior mesenteric artery supplies the midgut, and the inferior mesenteric artery supplies the hindgut.

4. The auricle (pinna) of the external ear is in- nervated by which of the following nerves? (A) CN V3 (B) CN V2 (C) CN XII (D) CN III (E) CN VIII

4. A. The auricle (pinna) of the external ear is innervated by cranial nerves V3 (mandibular divi- sion), VII, IX, and X; cervical nerves C2 and C3 also innervate the auricle.

4. The Doppler fetal heart rate is first audible in which of the following trimesters? (A) First trimester (B) Second trimester (C) Third trimester

4. A. The fetal heart rate is first audible in the first trimester at around week 12.

4. The most common site of a pharyngeal fistula is the (A) dorsal aspect of neck (B) anterior border of sternocleidomastoid muscle (C) superior mediastinum (D) midline close to the hyoid bone (E) base of the tongue

4. B. A pharyngeal fistula forms when pharyngeal pouch 2 and pharyngeal groove 2 persist. Therefore, these fistulas are found on the lateral aspect of the neck, usually along the anterior border of the sternocleidomastoid muscle.

4. A young insulin-dependent diabetic woman in her first pregnancy is concerned that her daily injection of insulin will cause a congenital malformation in her baby. What should the physician tell her? (A) Insulin is highly teratogenic; discontinue treatment (B) Insulin does not cross the placental membrane (C) Insulin crosses the placental membrane but is degraded rapidly (D) Insulin will benefit her baby by increasing glucose metabolism (E) Insulin crosses the placental membrane but is not teratogenic

4. B. Insulin, like all protein hormones, does not cross the placental membrane in significant amounts.

4. A young infant shows skin blisters over the entire body with generalized skin erosion. Pathology indicates a cleft between the epidermis and dermis. What is the most likely diagnosis? (A) Psoriasis (B) Junctional epidermolysis bullosa (C) Ichthyosis (D) Ehlers-Danlos syndrome (E) Type II oculocutaneous albinism

4. B. Junctional epidermolysis bullosa refers to a group of autosomal recessive disorders caused by a mutation in the gene for laminin 5.

4. During an ultrasound examination, numerous fractures of the long bones of the fetus are observed. This condition is called (A) achondroplasia (B) osteogenesis imperfecta (C) Marfan syndrome (D) cretinism (E) acromegaly

4. B. Osteogenesis imperfecta is a deficiency of type I collagen and results in spontaneous fractures of fetal bones and blue sclera of the eye.

4. Where does the blastocyst normally implant? (A) Functional layer of the cervix (B) Functional layer of the endometrium (C) Basal layer of the endometrium (D) Myometrium (E) Perimetrium

4. B. The blastocyst implants in the functional layer of the uterine endometrium. The uterus is composed of the perimetrium, myometrium, and endometrium. Two layers are identified within the endometrium: (1) the functional layer, which is sloughed off at menstruation, and (2) the basal layer, which is retained at menstruation and serves as the source of regeneration of the functional layer. During the progestational phase of the menstrual cycle, the functional layer undergoes dramatic changes; uterine glands enlarge and vascularity increases in preparation for blastocyst implantation.

4. The most common atrial septal defect (ASD) seen clinically is (A) common atrium (B) foramen secundum defect (C) premature closure of the foramen ovale (D) persistent truncus arteriosus (E) probe patency of the foramen ovale

4. B. The most common ASD is foramen secundum defect, which is caused by excessive resorp- tion of the septum primum or the septum secundum. This results in an opening between the atria (patent foramen ovale). Some of these defects may remain undiagnosed and may be toler- ated for a long time (up to age 30 years before the person presents clinically).

4. The transitional epithelium lining the urinary bladder is derived from (A) ectoderm (B) endoderm (C) mesoderm (D) endoderm and mesoderm (E) neural crest cells

4. B. The transitional epithelium lining the urinary bladder is derived from endoderm because the urinary bladder develops from the upper end of the urogenital sinus. The origin of the urogenital sinus can be traced back to the gut tube, which is lined by endoderm.

4. Which of the following is a parasite found in cats? (A) Treponema pallidum (B) Toxoplasma gondii (C) Rubella virus (D) Cytomegalovirus (E) Varicella zoster virus

4. B. Toxoplasma gondii is a protozoan parasite found in cats and may be transmitted to the fetus transplacentally.

4. The biceps femoris muscle develops from which of the following? (A) Hypomere (B) Epimere (C) Anterior condensation (D) Posterior condensation (E) Preotic somites

4. C. Because the biceps femoris muscle is a flexor of the leg, it develops from the anterior condensation of myotomic mesoderm.

4. The prochordal plate marks the site of the future (A) umbilical cord (B) heart (C) mouth (D) anus (E) nose

4. C. The prochordal plate is a circular, midline thickening of hypoblast cells that are firmly attached to the overlying epiblast cells. The plate will eventually develop into a membrane called the oropharyngeal membrane at the site of the future mouth. It is interesting to note that at this early stage of development the cranial versus caudal region of the embryo is established by the prochordal plate, and since the prochordal plate is located in the midline, bilateral symmetry is also established.

4. A structure found within the adult female pelvis formed from the gubernaculum is the (A) broad ligament (B) suspensory ligament of the ovary (C) round ligament of the uterus (D) medial umbilical ligament (E) median umbilical ligament

4. C. The round ligament of the uterus and the ovarian ligament both form from the gubernaculum.

4. Somites may differentiate into which of the following? (A) Urogenital ridge (B) Kidneys (C) Notochord (D) Epimeric and hypomeric muscles (E) Epithelial lining of the gastrointestinal tract

4. D. Approximately 35 pairs of somites form. They are derived from a specific subdivision of intraembryonic mesoderm called paraxial mesoderm. Somites differentiate into the components called sclerotome (cartilage and bone of the vertebral column), myotome (epimeric and hypomeric muscle), and dermatome (dermis and subcutaneous area of skin).

4. An 8-day-old boy presents with a history of complete loss of breath at times and of turning blue on a number of occasions. If the baby is placed in an upright or sitting position, his breathing improves. Physical examination reveals an unusually flat stomach when the newborn is lying down; auscultation demonstrates no breath sounds on the left side of the thorax. What is the diagnosis? (A) Physiological umbilical herniation (B) Esophageal hiatal hernia (C) Tetralogy of Fallot (D) Congenital diaphragmatic hernia (E) Tricuspid atresia

4. D. Loss of breath and cyanosis result from pulmonary hypoplasia associated with congenital diaphragmatic hernia. Placing the baby in an upright position will reduce the hernia somewhat and ease the pressure on the lungs, thereby increasing the baby's comfort. The baby's stomach is flat (instead of the plump belly of a normal newborn) because the abdominal viscera have herniated into the thorax. Auscultation reveals no breath sounds on the left side because of pulmonary hypoplasia.

4. Which of the following muscles will the posterior divisions of the lumbosacral plexus innervate? (A) Semitendinosus (B) Semimembranosus (C) Long head of biceps femoris (D) Rectus femoris (E) Gastrocnemius

4. D. One of the nerves that form from the posterior divisions of the lumbosacral plexus is the femoral nerve. Posterior divisions of the lumbosacral plexus will innervate muscles derived from the posterior condensation (extensors). Rectus femoris muscle is an extensor at the knee joint. Note that rectus femoris muscle and the femoral nerve are related embryologically to the posterior condensation and posterior divisions even though in the adult they are located anterior. This occurs because of the medial rotation of 90°.

4. The epithelium of the ciliary body is derived from (A) ectoderm (B) mesoderm (C) endoderm (D) neuroectoderm (E) neural crest cells

4. D. The ciliary body is derived from the anterior two layers of the optic cup (neuroectoderm), which form the epithelium, and from an anterior extension of the choroid (mesoderm).

4. Smooth muscle, connective tissue, and carti- lage of primary bronchi are derived from which of the following sources? (A) Neuroectoderm (B) Endoderm (C) Ectoderm (D) Visceral mesoderm (E) Mesoderm of pharyngeal arches 4 and 6

4. D. The epithelium of primary bronchi is derived from endoderm; the other components are derived from visceral mesoderm.

4. Which of the following represents the general visceral efferent (GVE) column of the pons? (A) The cerebellum (B) The spinal trigeminal nucleus (C) The principal trigeminal nucleus (D) The superior salivatory nucleus (E) The pontine nuclei

4. D. The superior salivatory nucleus represents the general visceral efferent (GVE) column of the pons. All somatic and visceral motor nuclei are derived from the basal plate. The cerebellum and pontine nuclei and the sensory nuclei of cranial nerves are derivatives of the alar plate.

4. Which of the following muscles will the lateral cord of the brachial plexus innervate? (A) Triceps (B) Supinator (C) Extensor carpi ulnaris (D) Extensor digitorum (E) Biceps brachii

4. E. One of the nerves that form from the lateral cord of the brachial plexus is the musculocuta- neous nerve. The musculocutaneous nerve will innervate muscles derived from the anterior condensation (flexors). The biceps brachii muscle is a flexor at the elbow joint. Note that the biceps brachii muscle and the musculocutaneous nerve are related embryologically to the anterior condensation and anterior divisions (which form the lateral cord) and in the adult are located anterior. This occurs because of the lateral rotation of 90°.

4. The most common cause of testicular feminization syndrome is (A) a 45, XO genotype (B) a 47, XXY genotype (C) inadequate production of testosterone and MIF (D) congenital adrenal hyperplasia (E) lack of androgen receptors

4. E. The most common cause of testicular feminization syndrome is the lack of androgen receptors in the urogenital folds and labioscrotal swellings. Because these tissues lack androgen receptors, they are blind or unresponsive to androgens. Consequently, these tissues develop into female external genitalia even though the fetus has a 46, XY genotype.

5. During week 4, the developing diaphragm is located at (A) C3, C4, C5 (B) T3, T4, T5 (C) T8, T9, T10 (D) L1, L2, L3 (E) L4, L5, L6

5. A. Although it may seem unusual, the adult diaphragm has its embryological beginning at the cervical level (C3, C4, C5). Nerve roots from C3, C4, and C5 enter the developing diaphragm, bringing both motor and sensory innervation. With the subsequent rapid growth of the neural tube, there is an apparent descent of the diaphragm to its adult levels (thoracic and lumbar). However, the diaphragm retains its innervation from C3, C4, and C5, which explains the unusually long phrenic nerves.

5. In the male, failure of the urethral folds to fuse completely results in (A) hypospadias (B) epispadias (C) cryptorchidism (D) congenital inguinal hernia (E) hydrocele

5. A. Failure of the urethral folds to fuse completely results in the external urethral orifice opening onto the ventral surface of the penis, a condition known as hypospadias.

5. During weeks 6-8, the lower limb bud will rotate (A) Medially 90° (B) Laterally 90° (C) Medially 180° (D) Laterally 180° (E) No rotation occurs

5. A. The lower limb bud will rotate medially 90° so that the knee points anteriorly.

5. Warfarin falls into which category of drugs? (A) Category X drugs (B) Category D drugs

5. A. Warfarin is a Category X drug.

5. The lungs become capable of respiration in which of the following trimesters? (A) First trimester (B) Second trimester (C) Third trimester

5. B. The lungs become capable of respiration at weeks 25-27 in the second trimester.

5. During weeks 6-8, the upper limb will rotate (A) Medially 90° (B) Laterally 90° (C) Medially 180° (D) Laterally 180° (E) No rotation occurs

5. B. The upper limb rotates laterally 90° so that the elbows point posteriorly.

5. Intermediate mesoderm will give rise to the (A) neural tube (B) heart (C) kidneys and gonads (D) somites (E) notochord

5. C. Intermediate mesoderm is a subdivision of intraembryonic mesoderm that forms a longitu- dinal dorsal ridge called the urogenital ridge from which the kidneys and gonads develop.

5. The external germinal layer of the cerebel- lum gives rise to which of the following? (A) Outer stellate cells (B) Purkinje cells (C) Granule cells (D) Basket cells (E) Giant cells of Betz

5. C. New evidence documents that the external granular layer gives rise only to the granule cells of the internal granular layer and not to the basket (inner stellate) or stellate (outer stellate) neurons, as was long thought. The giant cells of Betz are found in the cerebral cortex.

5. The most common type of anorectal malformation is (A) imperforate anus (B) anal agenesis (C) anorectal agenesis (D) rectal atresia (E) colonic aganglionosis

5. C. The most common type of malformation involving the anal canal and rectum is anorectal agenesis, in which the rectum ends as a blind sac above the puborectalis muscle. The anal canal may form normally but does not connect with the rectum. This malformation is accompanied by various fistulas.

5. What is a normal amount of amniotic fluid at term? (A) 50 mL (B) 500 mL (C) 1000 mL (D) 1500 mL (E) 2000 mL

5. C. The normal amount of amniotic fluid at term is 1000 mL. However, the amount of amniotic fluid at various stages of pregnancy can be indicative of congenital malformations. Oligohy- dramnios (400 mL in late pregnancy) may be indicative of renal agenesis. Polyhydramnios (2000 mL in late pregnancy) may be indicative of either anencephaly or esophageal atresia.

5. The transitional epithelium lining the ureter is derived from (A) ectoderm (B) endoderm (C) mesoderm (D) endoderm and mesoderm (E) neural crest cells

5. C. The transitional epithelium lining the ureter is derived from mesoderm because the ureter develops from the ureteric bud. The ureteric bud is a diverticulum from the mesonephric duct whose origin can be traced back to the intermediate mesoderm.

5. The labia minora arise embryologically from which of the following structures? (A) Phallus (B) Labioscrotal swellings (C) Sinovaginal bulbs (D) Urogenital folds (E) Paramesonephric duct

5. D. In the female, the urogenital folds remain unfused and form the labia minora.

5. Hyperoxygenation of premature infants may result in (A) congenital glaucoma (B) microphthalmia (C) coloboma (D) retrolental fibroplasia (E) persistent pupillary membrane

5. D. Retrolental fibroplasia results from hyperoxygenation of premature infants. In premature infants, high oxygen concentration results in vaso-obliteration of the terminal arterioles, leading to hemorrhage and infarction of the retina. This phenomenon is peculiar to the incompletely vascularized peripheral retina.

5. Which of the following are components of the definitive chorion? (A) Extraembryonic somatic mesoderm and epiblast (B) Extraembryonic somatic mesoderm and cytotrophoblast (C) Extraembryonic somatic mesoderm and syncytiotrophoblast (D) Extraembryonic somatic mesoderm, cytotrophoblast, and syncytiotrophoblast (E) Extraembryonic visceral mesoderm, cytotrophoblast, and syncytiotrophoblast

5. D. The definitive chorion consists of three components: extraembryonic somatic mesoderm, cytotrophoblast, and syncytiotrophoblast. The chorion defines the chorionic cavity in which the embryoblast is suspended and is vital in the formation of the placenta.

5. The stapedius muscle, which moves the sta- pes ossicle, is innervated by (A) CN V3 (B) CN XII (C) CN III (D) CN VII (E) cervical nerves C2 and C3

5. D. The stapes is innervated by CN VII.

5. What is the most common congenital malformation of the head and neck region? (A) Anterior cleft palate (B) Posterior cleft palate (C) Thyroglossal duct cyst (D) Unilateral cleft lip (E) Ankyloglossia

5. D. Unilateral cleft lip is the most common congenital malformation of the head and neck. Cleft lip occurs when the maxillary prominences fail to fuse with the medial nasal prominences and when the underlying somitomeric mesoderm and neural crest fail to proliferate, resulting in a persistent labial groove. Cleft lip occurs in 1 of 900 births and may be unilateral or bilateral.

5. A female newborn presents with a square- shaped skull with a short occipital-frontal diameter. Premature closure of which of the following sutures is the most likely cause of this finding? (A) Sphenofrontal (B) Sphenoparietal (C) Lambdoidal (D) Sagittal (E) Coronal

5. E. Brachycephaly is the premature closure of the coronal sutures, which leads to a square-shaped skull with a short occipital-frontal diameter.

5. Which of the following events is involved in the cleavage of the zygote during week 1 of development? (A) A series of meiotic divisions forming blastomeres (B) Production of highly differentiated blastomeres (C) An increased cytoplasmic content of blastomeres (D) An increase in size of blastomeres (E) A decrease in size of blastomeres

5. E. Cleavage is a series of mitotic divisions by which the large amount of zygote cytoplasm is successively partitioned among the newly formed blastomeres. Although the number of blas- tomeres increases during cleavage, the size of individual blastomeres decreases until they re- semble adult cells in size.

5. The ventral surface of the adult heart as seen on gross examination or radiography is comprised primarily of the (A) left atrium (B) left ventricle (C) inferior vena cava (D) bulbus cordis (E) right ventricle

5. E. During embryological formation of the heart, the arterial and venous ends of the heart tube are fixed in place. As further growth continues, the heart tube folds to the right. This greatly con- tributes to the ventral surface of the adult heart being comprised primarily of the right ventricle. The definitive anatomical orientation of the adult heart within the thorax is not at all similar to the strong image we have in our minds of the classic Valentine's Day heart.

5. The administration of which of the following agents may result in discoloration of both deciduous and permanent teeth? (A) Cephalosporin (B) Chloramphenicol (C) Erythromycin (D) Penicillin (E) Tetracycline

5. E. Tetracyclines are bound to calcium in newly formed teeth both in utero and in young children. They may cause discoloration and enamel dysplasia.

5. Components of the blood-air barrier in the lung are derived from which of the following sources? (A) Ectoderm only (B) Visceral mesoderm only (C) Visceral mesoderm and ectoderm (D) Endoderm and ectoderm (E) Visceral mesoderm and endoderm

5. E. The blood-air barrier comprises the structures through which gaseous exchange occurs between air in alveoli and blood in pulmonary capillaries. The attenuated pulmonary epithelium (type I pneumocytes) is derived from endoderm. The simple, squamous epithelium (endothelium) lining pulmonary capillaries is derived from visceral mesoderm.

6. Which pharyngeal arch is associated with Treacher Collins syndrome? (A) Pharyngeal arch 1 (B) Pharyngeal arch 2 (C) Pharyngeal arch 3 (D) Pharyngeal arch 4 (E) Pharyngeal arch 6

6. A. First arch syndrome results from abnormal development of pharyngeal arch 1 due to a lack of migration of neural crest cells. Treacher Collins syndrome is associated with underdevelop- ment of the zygomatic bone, down-slanting palpebral fissures, and deformed lower eyelids and external ears.

6. Which of the following structures produces progesterone late in pregnancy? (A) Placenta (B) Corpus luteum (C) Syncytiotrophoblast (D) Fetal adenohypophysis (E) Maternal liver

6. A. Progesterone is a steroid hormone that is produced by the placenta up until birth. The corpus luteum also produces progesterone, but only until week 8 of pregnancy.

6. An apparently healthy newborn with a hardy appetite has begun feedings with formula. When she is laid down in the crib after feeding, she experiences projectile vomiting. Which of the following conditions is a probable cause of this vomiting? (A) Physiological umbilical herniation (B) Esophageal hiatal hernia (C) Tetralogy of Fallot (D) Congenital diaphragmatic hernia (E) Tracheoesophageal fistula

6. B. An esophageal hiatal hernia is a herniation of the stomach through the esophageal hiatus into the pleural cavity. This compromises the esophagogastric sphincter so that stomach contents can easily reflux into the esophagus. The combination of a full stomach after feeding and lying down in the crib will cause vomiting in this newborn.

6. Which of the following does not pass through the primitive umbilical ring? (A) Allantois (B) Amnion (C) Yolk sac (D) Connecting stalk (E) Space connecting the intraembryonic and extraembryonic coeloms

6. B. The amnion does not pass through the primitive umbilical ring. As craniocaudal folding occurs, the amnion becomes the outer covering of the umbilical cord.

6. The simple columnar or cuboidal epithelium lining the extrahepatic biliary ducts is derived from (A) mesoderm (B) endoderm (C) ectoderm (D) neuroectoderm (E) neural crest cells

6. B. The epithelium lining the extrahepatic biliary ducts is derived from endoderm. The intrahe- patic biliary ducts are also derived from endoderm.

6. Which of the following structures must degenerate for blastocyst implantation to occur? (A) Endometrium in progestational phase (B) Zona pellucida (C) Syncytiotrophoblast (D) Cytotrophoblast (E) Functional layer of the endometrium

6. B. The zona pellucida must degenerate for implantation to occur. Early cleavage states of the blastula are surrounded by a zona pellucida, which prevents implantation in the uterine tube.

6. Valium falls into which category of drugs? (A) Category X drugs (B) Category D drugs

6. B. Valium is a Category D drug.

6. A 16-year-old girl presents on May 10 in obvious emotional distress. On questioning, she relates that on May 1 she experienced sexual intercourse for the first time, without using any means of birth control. Most of her anxiety stems from her fear of pregnancy. What should the physician do to alleviate her fear? (A) Prescribe diazepam and wait to see if she misses her next menstrual period (B) Use ultrasonography to document pregnancy (C) Order a laboratory assay for serum hCG (D) Order a laboratory assay for serum progesterone (E) Prescribe diethylstilbestrol ("morning-after pill")

6. C. Human chorionic gonadotropin (hCG) can be assayed in maternal serum at day 8 of devel- opment and in urine at day 10. If this teenager is pregnant, the blastocyst would be in week 2 of development (day 10). Laboratory assay of hCG in either the serum or urine can be completed; however, serum hCG might be more reliable. It is important to note that if she is pregnant, she will not miss a menstrual period until May 15, at which time the embryo will be entering week 3 of development.

6. The uterine tubes of the adult female are derived embryologically from which of the following? (A) Mesonephric duct (B) Mesonephric tubules (C) Paramesonephric duct (D) Paramesonephric tubules (E) Uterovaginal primordium

6. C. The cranial portion of the paramesonephric ducts forms the uterine tubes.

6. The Y chromosome carries a gene on its short arm that codes for (A) testosterone (B) MIF (C) testes-determining factor (TDF) (D) progesterone (E) estrogen

6. C. The gene product that is coded on the short arm of the Y chromosome is called the TDF.

6. The podocytes of Bowman capsule are derived from (A) ectoderm (B) endoderm (C) mesoderm (D) endoderm and mesoderm (E) neural crest cells

6. C. The podocytes of Bowman capsule develop from the metanephric vesicles, which are of mesodermal origin.

6. The utricular portion of the otic vesicle gives rise to the (A) ductus reuniens (B) cochlear duct (C) endolymphatic sac (D) scala vestibuli (E) scala tympani

6. C. The utricular region of the otic vesicle gives rise to the endolymphatic sac and duct and semicircular ducts.

6. Which of the following statements best describes the pathogenesis of hydranencephaly? (A) Results from failure of midline cleavage of the embryonic forebrain (B) Results from atresia of the outlet foramina of the fourth ventricle (C) Results from blockage of the cerebral aqueduct (D) Results from internal carotid artery occlusion (E) Results from failure of the anterior neuro- pore to close

6. D. Hydranencephaly consists of huge intracerebral cavitation resulting from infarction in the territory of the internal carotid artery.

6. The left recurrent laryngeal nerve recurs around the (A) left primary bronchus (B) left subclavian artery (C) left subclavian vein (D) ductus arteriosus (E) left common carotid artery

6. D. The left recurrent laryngeal nerve recurs around the ductus arteriosus (ligamentum arteriosus in the adult). Early in embryological development, both the right and left recurrent laryngeal nerves hook (recur) around aortic arch 6. The left aortic arch 6 persists as the ductus arteriosus.

6. The respiratory diverticulum initially is in open communication with the primitive fore- gut. Which of the following embryonic struc- tures is responsible for separating these two structures? (A) Laryngotracheal groove (B) Posterior esophageal folds (C) Laryngotracheal diverticulum (D) Tracheoesophageal septum (E) Bronchopulmonary segment

6. D. When the tracheoesophageal folds fuse in the midline, they form the tracheoesophageal septum. This septum is responsible for separating the adult trachea ventrally from the esophagus dorsally.

6. The optic nerve is a tract of the diencephalons that is not completely myelinated until (A) 5 years after birth (B) 2 years after birth (C) 1 year after birth (D) 3 weeks after birth (E) 3 months after birth

6. E. The axons of the optic nerve are not completely myelinated until 3 months after birth. Myelinated axons are normally not found in the retina. The optic nerve is not a true periph- eral nerve but a tract of the diencephalon; when severed, the optic nerve does not regener- ate. Myelination in the central nervous system (CNS) is accomplished by oligodendrocytes; oligodendrocytes are not found in the retina.

6. The developing embryo has a distinct human appearance by the end of (A) week 4 (B) week 5 (C) week 6 (D) week 7 (E) week 8

6. E. The embryo starts the embryonic period as a two-dimensional disk and ends as a three-dimensional cylinder. This dramatic change in geometry is caused by formation of all the major organ systems. As the organ systems gradually develop during the embryonic period, the embryo appears more and more human-like; it has a distinct human appearance at the end of week 8.

7. The hyaloid canal is found in the (A) vitreous body (B) choroid (C) optic stalk (D) ciliary body (E) intraretinal space

7. A. The hyaloid canal (Cloquet's canal) is found in the vitreous body. In early development, a hyaloid artery passes through the vitreous body to perfuse the developing lens; in the late fetal period, this artery is obliterated to form the hyaloid canal.

7. The saccular portion of the otic vesicle gives rise to the (A) organ of Corti (B) endolymphatic duct (C) superior semicircular canal (D) crus commune nonampullare (E) lateral semicircular canal

7. A. The saccular region of the otic vesicle gives rise to the cochlear duct, which houses the spiral organ of Corti.

7. Collapse of bronchi caused by failure of bronchial cartilage development is indica- tive of which one of the following congenital malformations? (A) Congenital bronchial cysts (B) Congenital neonatal emphysema (C) Tracheoesophageal fistula (D) Hyaline membrane disease (E) Pulmonary hypoplasia

7. B. Congenital neonatal emphysema is a malformation involving the bronchi. One or more lobes of the lungs are overdistended with air because air can be inspired through collapsed bronchi but cannot be expired.

7. Bilateral cryptorchidism usually results in (A) impotence (B) sterility (C) male pseudo-intersexuality (D) female pseudo-intersexuality (E) testicular feminization syndrome

7. B. Sterility is a common result of bilateral cryptorchidism. When both testes fail to descend into the scrotum, the increased temperature to which they are exposed in the abdominal cavity will inhibit spermatogenesis.

7. The proximal convoluted tubules of the de- finitive adult kidney are derived from the (A) ureteric bud (B) metanephric vesicle (C) mesonephric duct (D) mesonephric tubules (E) pronephric tubules

7. B. The distal convoluted tubule, loop of Henle, proximal convoluted tubule, and Bowman capsule are all derived from the metanephric vesicle.

7. The lateral mesoderm is divided into two distinct layers by the formation of the (A) extraembryonic coelom (B) intraembryonic coelom (C) cardiogenic region (D) notochord (E) yolk sac

7. B. The lateral mesoderm is a subdivision of intraembryonic mesoderm and initially is a solid plate of mesoderm. The intraembryonic coelom forms in the middle of the lateral mesoderm, thereby dividing it into the intraembryonic somatic mesoderm and intraembryonic visceral mesoderm.

7. Which of the following is the origin of the mitochondrial DNA of all human adult cells? (A) Paternal only (B) Maternal only (C) A combination of paternal and maternal (D) Either paternal or maternal (E) Unknown origin

7. B. The mitochondrial DNA of all human adult cells is of maternal origin only. In human fertil- ization, the entire sperm enters the secondary oocyte cytoplasm. However, sperm mitochondria degenerate along with the sperm's tail. Therefore, only mitochondria present within the second- ary oocyte (maternal) remain in the fertilized zygote.

7. The anterior and posterior neuropores close during which week of embryonic development? (A) Week 2 (B) Week 3 (C) Week 4 (D) Week 5 (E) Week 6

7. C. The anterior and posterior neuropores close during week 4 of development—the anterior on day 25, the posterior on day 27. Failure of the anterior neuropore to close results in anencephaly; failure of the posterior neuropore to close results in myeloschisis.

7. Which of the three primary germ layers forms the histologically definitive endocardium of the adult heart? (A) Ectoderm (B) Endoderm (C) Mesoderm (D) Epiblast (E) Hypoblast

7. C. The entire cardiovascular system is of mesodermal origin.

7. During surgery for the removal of a thy- roid tumor, a number of small masses of glandular tissue are noted just lateral to the thyroid gland. Metastasis from the thyroid tumor is suspected, but histological analysis of a biopsy reveals parathyroid tissue and remnants of thymus. How can this finding be explained? (A) Tumor tissue has differentiated into nor- mal tissue (B) A parathyroid gland tumor is also present (C) Ectopic glandular tissue is commonly found in this region (D) The patient has DiGeorge syndrome (E) The glandular tissue is a result of a thyroglossal duct cyst

7. C. The parathyroid and thymus migrate in a caudal and medial direction during development; therefore, ectopic glandular tissue may be found anywhere along the migratory path.

7. Which of the following best describes the placental components of dizygotic twins? (A) One placenta, two amniotic sacs, one chorion (B) One placenta, two amniotic sacs, two chorions (C) Two placentas, two amniotic sacs, one chorion (D) Two placentas, two amniotic sacs, two chorions (E) One placenta, two amniotic sacs, two chorions

7. D. Dizygotic twins and 35% of monozygotic twins have two placentas, two amniotic sacs, and two chorions ("222").

7. Carcinoembryonic antigen (CEA) is an oncofetal antigen that is generally associated with which one of the following tumors? (A) Hepatoma (B) Germ cell tumor (C) Squamous cell carcinoma (D) Colorectal carcinoma (E) Teratocarcinoma

7. D. Oncofetal antigens are normally expressed during embryonic development, remain unexpressed in normal adult cells, but are re-expressed on transformation to malignant neoplastic tissue. CEA is associated with colorectal carcinoma.

7. A 4-day-old baby boy has not defecated since coming home from the hospital even though feeding has been normal without any excessive vomiting. Rectal examination reveals a normal anus, anal canal, and rectum. How- ever, a large fecal mass is found in the colon, and a large release of flatus and feces follows the rectal examination. Which of the following conditions would be suspected? (A) Imperforate anus (B) Anal agenesis (C) Anorectal agenesis (D) Rectal atresia (E) Colonic aganglionosis

7. E. This baby boy suffers from colonic aganglionosis, or Hirschsprung disease, which results in the retention of fecal material, causing the normal colon to enlarge. The retention of fecal material results from a lack of peristalsis in the narrow segment of colon distal to the enlarged colon. A biopsy of the narrow segment of colon would reveal the absence of parasympathetic ganglion cells in the myenteric plexus caused by failure of neural crest migration.

8. Very often the first indication a woman has that she is pregnant is a missed menstrual period. In which week of embryonic development will a woman experience her first missed menstrual period? (A) Start of week 3 (B) Start of week 4 (C) Start of week 5 (D) Start of week 8 (E) End of week 8

8. A. Given a regular 28-day menstrual cycle, a woman who starts menses on, say, February 1 will ovulate on February 14, and the secondary oocyte will be fertilized, if she becomes pregnant, within 24 hours. So, the zygote undergoes week 1 of development from February 15 to 21. Week 2 of development is from February 22 to 28. On the next day, March 1, the woman would enter her next menstrual cycle if she were not pregnant, but because she is pregnant, she does not menstruate. Therefore, this first missed menstrual period corresponds with the start of week 3 of embryonic development. The embryonic period (week 3-week 8) is a time of high susceptibility to teratogens.

8. Which one of the following structures is derived from the midgut? (A) Appendix (B) Stomach (C) Liver (D) Pancreas (E) Sigmoid colon

8. A. The appendix is derived from the midgut. The midgut normally undergoes a 270° counter- clockwise rotation during development; malrotation of the midgut may result in the appendix lying in the upper part of the abdominal cavity, which may affect a diagnosis of appendicitis.

8. The trigone on the posterior wall of the urinary bladder is formed by the (A) incorporation of the lower end of the mesonephric ducts (B) incorporation of the lower end of the pronephric ducts (C) incorporation of the metanephric blastema (D) incorporation of the mesonephric tubules (E) incorporation of the pronephric tubules

8. A. The lower ends of the mesonephric ducts are incorporated into the posterior wall of the urinary bladder. The mesonephric ducts contribute to the connective tissue component of the posterior wall at the trigone. It is generally believed that the transitional epithelium lining the entire bladder (even the trigone) is of endodermal origin.

8. The tubotympanic recess gives rise to (A) a conduit that interconnects the middle ear and the nasopharynx (B) the external auditory meatus (C) the internal auditory meatus (D) the facial canal (E) a conduit that interconnects the perilym- phatic space with the subarachnoid space

8. A. The tubotympanic recess is derived from pharyngeal pouch 1. It gives rise to the tympanic cavity and the auditory (eustachian) tube; the auditory tube interconnects the tympanic cavity with the nasopharynx.

8. A 17-year-old girl presents with a complaint of amenorrhea. Physical examination reveals good breast development and normal amount of pubic hair. A rudimentary vagina and a mobile mass within both the right and left labia majora are found on pelvic examination. Ultrasound reveals the absence of a uterus. What is the diagnosis? (A) Testicular feminization syndrome (B) Gonadal dysgenesis (C) Cryptorchidism (D) Female pseudo-intersexuality (E) Hypospadias

8. A. This is a classic case of testicular feminization syndrome. A karyotype analysis would reveal that this normal-appearing 17-year-old girl actually has a 46, XY genotype. The mobile masses within the right and left labia majora are the testes and should be surgically removed because this tissue has a propensity toward malignant tumor formation. The most common cause of this syndrome is a lack of androgen receptors in the phallus, urogenital folds, and labioscrotal swellings.

8. Pulmonary hypoplasia is commonly associ- ated with which condition? (A) Hyaline membrane disease (B) Diaphragmatic hernia (C) Tracheoesophageal fistula (D) Congenital bronchial cysts (E) Congenital neonatal emphysema

8. B. During normal development, a space is provided for the prolific growth of the bronchial buds in a lateral and caudal direction. This space, which is part of the intraembryonic coelom, is called the primitive pleural cavity. If this space is reduced by herniation of abdominal viscera, lung development will be severely compromised.

8. A 26-year-old pregnant woman experiences repeated episodes of bright red vaginal bleed- ing at week 28, week 32, and week 34 of preg- nancy. The bleeding spontaneously subsided each time. Use of ultrasound shows that the placenta is located in the lower right portion of the uterus over the internal os. What is the diagnosis? (A) Hydatidiform mole (B) Vasa previa (C) Placenta previa (D) Placental abruption (E) Premature rupture of the amniochorionic membrane

8. C. A placenta implanted in the lower part of the uterus near the internal os is called placenta previa. The repeated episodes of bright-red vaginal bleeding are caused by the gradual dilation of the uterus in the later stages of pregnancy. As the uterus dilates, spiral arteries and veins supplying the placenta are ruptured. The mother may bleed to death, and the fetus is placed in jeopardy because of the compromised maternal blood flow.

8. Aqueous humor is produced by the (A) choroid plexus (B) trabecular meshwork (C) ciliary processes (D) vitreous body (E) lens vesicle

8. C. Aqueous humor is produced by the ciliary processes of the ciliary body. It flows from the posterior chamber, through the pupil, into the anterior chamber, and finally to the canal of Schlemm, which empties into the extraocular veins.

8. At birth the conus medullaris is found at which vertebral level? (A) T12 (B) L1 (C) L3 (D) S1 (E) S4

8. C. At birth, the conus medullaris extends to L3, and in the adult it extends to the L1-L2 inter- space. At 8 weeks, the spinal cord extends the entire length of the vertebral canal.

8. A newborn presents with midfacial and mandibular hypoplasia, defects of the first pharyngeal arch consistent with the diagnosis of Treacher Collins syndrome. What structure would most likely be involved with the syndrome? (A) Hyoid bone (B) Stapes (C) Malleus (D) Thyroid gland (E) Inferior parathyroid gland

8. C. The malleus is the only structure on this list derived from the neural crest of the first pharyngeal arch.

8. Individual blastomeres were isolated from a blastula at the 4-cell stage. Each blastomere was cultured in vitro to the blastocyst stage and in- dividually implanted into four pseudopregnant foster mothers. Which of the following would you expect to observe 9 months later? (A) Birth of one baby (B) Birth of four genetically different babies (C) Birth of four genetically identical babies (D) Birth of four grotesquely deformed babies (E) No births

8. C. This scenario would result in four genetically identical children. Blastomeres at the 4- to 8-cell stage are totipotent, that is, capable of forming an entire embryo. Since blastomeres arise by mitosis of the same cell (zygote), they are genetically identical. This phenomenon is impor- tant in explaining monozygotic (identical) twins. About 30% of monozygotic twins arise by early separation of blastomeres. The remaining 70% originate at the end of week 1 of development by a splitting of the inner cell mass.

8. Which of the following is responsible for the proper alignment of the atrioventricular canal and the conoventricular canal? (A) Lateral folding of the embryo (B) Craniocaudal folding of the embryo (C) Programmed cell migration (D) Formation of the aorticopulmonary septum (E) Dextral looping

8. E. Dextral looping aligns these two canals through early looping, convergence, wedging, and repositioning. This is especially important in correcting the unusual blood flow pattern in the primitive heart tube where venous blood flows into the left ventricle prior to the right ventricle.

9. A 19-year-old woman in week 32 of a com- plication-free pregnancy is rushed to the emer- gency department because of profuse vaginal bleeding. The bleeding subsides, but afterward no fetal heart sounds can be heard, indicating intrauterine fetal death. The woman goes into labor and delivers a stillborn infant. On exami- nation of the afterbirth, a velamentous placenta is detected. Although not much can be done at this point, what is the diagnosis? (A) Placenta previa (B) Vasa previa (C) Hydatidiform mole (D) Premature rupture of the amniochorionic membrane (E) Amniotic band syndrome

9. B. A velamentous placenta occurs when umbilical blood vessels abnormally travel through the amniochorionic membrane before reaching the placenta proper. If the vessels cross the internal os, a serious condition called vasa previa exists. As the fetus grows during pregnancy and the amniochorionic membrane stretches, the umbilical vessels may rupture. When that happens, the fetus will bleed to death. The mother is in no danger of bleeding to death in vasa previa because only the umbilical vessels rupture.

9. A female newborn was found to have a large midline tumor in the lower sacral area, which was diagnosed as a sacrococcygeal tumor. Which of the following courses of treatment is recommended for this child? (A) Immediate chemotherapy and radiation treatment (B) Surgical removal by age 6 months (C) Surgical removal at age 4-5 years (D) Surgical removal at age 13-15 years (E) No treatment because this tumor normally regresses with age

9. B. The preponderance of sacrococcygeal tumors is found in female newborns. Because these tumors develop from pluripotent cells of primitive streak origin, malignancy is of great concern, and the tumor should be surgically removed by age 6 months. Occasionally, these tumors may recur after surgery, demonstrating malignant properties.

9. Which of the following structures is derived from the telencephalon? (A) Pineal gland (B) Hypothalamus (C) Hippocampus (D) Optic nerve (CN II) (E) Globus pallidus

9. C. The hippocampus develops from the telencephalon. The pineal gland, hypothalamus, CN II, and the globus pallidus are derived from the diencephalon.

9. Perilymph enters the subarachnoid space via the (A) cochlear duct (B) ductus reuniens (C) perilymphatic duct (D) vestibular aqueduct (E) utriculosaccular duct

9. C. The perilymph enters the subarachnoid space of the posterior cranial fossa via the cochlear aqueduct, which contains the perilymphatic duct.

9. A 6-year-old girl presents with a large abdominal mass just superior to the pubic symphysis. The mass is tender when palpated and fixed in location. During surgery, a fluid-filled mass is noted connected to the umbilicus superiorly and to the urinary bladder inferiorly. What is the diagnosis? (A) Pelvic kidney (B) Horseshoe kidney (C) Polycystic disease of the kidney (D) Urachal cyst (E) Exstrophy of the bladder

9. D. A urachal cyst or sinus forms from a remnant of the allantois and is found along the midline on a path from the umbilicus to the apex of the urinary bladder. The epithelium lining the cyst produces secretions that gradually fill the remnant with fluid. Very rarely, the entire allantois persists, forming a fistula that is patent from the urinary bladder to the exterior at the umbilicus.

9. Aqueous humor enters the venous circulation via (A) arachnoid villi (B) scleral canal (C) hyaloid canal (D) canal of Schlemm (E) Cloquet's canal

9. D. Aqueous humor enters the venous circulation via the canal of Schlemm. Blockage of this canal results in increased intraocular pressure (glaucoma).

9. The hepatic sinusoids that can be observed histologically in an adult liver are derived from the (A) supracardinal veins (B) anterior cardinal veins (C) posterior cardinal veins (D) vitelline veins (E) subcardinal veins

9. D. Because of the location of the vitelline veins and the tremendous growth of the developing liver (hepatic diverticulum), the vitelline veins are surrounded by the liver and give rise to the hepatic sinusoids. The umbilical veins also contribute to the hepatic sinusoidal network.

9. A 3-month-old baby girl presents with a swollen umbilicus that has failed to heal nor- mally. The umbilicus drains secretions, and there is passage of fecal material through the umbilicus at times. What is the most likely diagnosis? (A) Omphalocele (B) Gastroschisis (C) Anal agenesis (D) Ileal diverticulum (E) Intestinal stenosis

9. D. This baby girl has an ileal diverticulum (Meckel diverticulum), which occurs when a rem- nant of the vitelline duct persists. In this case, a fistula is present by which contents of the ileum can be discharged onto the surface of the skin.

9. Embryonic carcinoma (EC) cells were isolated from a yellow-coated mouse with a teratocarcinoma. The EC cells were then microinjected into the inner cell mass of a blastocyst isolated from a black-coated mouse. The blastocyst was subsequently implanted into the uterus of a white-coated foster mouse. Which of the following would be observed after full-term pregnancy? (A) A yellow-coated offspring (B) A black-coated offspring (C) A white-coated offspring (D) A yellow- and black-coated offspring (E) A yellow- and white-coated offspring

9. D. This scenario would result in a yellow- and black-coated offspring. Because EC cells and inner cell mass cells have very similar biochemical characteristics, they readily mix with each other, and development proceeds unencumbered. Because the mixture contains cells with yellow-coat genotype and black-coat genotype, offspring with coats of two colors (yellow and black) will be produced. The offspring are known as mosaic mice.

9. Development of which of the following is the first sign of respiratory system development? (A) Tracheoesophageal septum (B) Hypobranchial eminence (C) Primitive foregut (D) Tracheoesophageal fistula (E) Respiratory diverticulum

9. E. Development of the respiratory system begins in week 4; the first sign of development is for- mation of the respiratory diverticulum in the ventral wall of the primitive foregut.

A

A


संबंधित स्टडी सेट्स

Delgado BUSG 275 (Internship) Final

View Set

Coordinating, Correlative and Subordinate Conjunctions

View Set

20th Century World History: World War I

View Set

Contract 1: Chapter 1 (Introduction) and 2 (Promise and Assent)

View Set